www.vorhilfe.de
Vorhilfe

Kostenlose Kommunikationsplattform für gegenseitige Hilfestellungen.
Hallo Gast!einloggen | registrieren ]
Startseite · Forum · Wissen · Kurse · Mitglieder · Team · Impressum
Forenbaum
^ Forenbaum
Status Vorhilfe
  Status Geisteswiss.
    Status Erdkunde
    Status Geschichte
    Status Jura
    Status Musik/Kunst
    Status Pädagogik
    Status Philosophie
    Status Politik/Wirtschaft
    Status Psychologie
    Status Religion
    Status Sozialwissenschaften
  Status Informatik
    Status Schule
    Status Hochschule
    Status Info-Training
    Status Wettbewerbe
    Status Praxis
    Status Internes IR
  Status Ingenieurwiss.
    Status Bauingenieurwesen
    Status Elektrotechnik
    Status Maschinenbau
    Status Materialwissenschaft
    Status Regelungstechnik
    Status Signaltheorie
    Status Sonstiges
    Status Technik
  Status Mathe
    Status Schulmathe
    Status Hochschulmathe
    Status Mathe-Vorkurse
    Status Mathe-Software
  Status Naturwiss.
    Status Astronomie
    Status Biologie
    Status Chemie
    Status Geowissenschaften
    Status Medizin
    Status Physik
    Status Sport
  Status Sonstiges / Diverses
  Status Sprachen
    Status Deutsch
    Status Englisch
    Status Französisch
    Status Griechisch
    Status Latein
    Status Russisch
    Status Spanisch
    Status Vorkurse
    Status Sonstiges (Sprachen)
  Status Neuerdings
  Status Internes VH
    Status Café VH
    Status Verbesserungen
    Status Benutzerbetreuung
    Status Plenum
    Status Datenbank-Forum
    Status Test-Forum
    Status Fragwürdige Inhalte
    Status VH e.V.

Gezeigt werden alle Foren bis zur Tiefe 2

Navigation
 Startseite...
 Neuerdings beta neu
 Forum...
 vorwissen...
 vorkurse...
 Werkzeuge...
 Nachhilfevermittlung beta...
 Online-Spiele beta
 Suchen
 Verein...
 Impressum
Das Projekt
Server und Internetanbindung werden durch Spenden finanziert.
Organisiert wird das Projekt von unserem Koordinatorenteam.
Hunderte Mitglieder helfen ehrenamtlich in unseren moderierten Foren.
Anbieter der Seite ist der gemeinnützige Verein "Vorhilfe.de e.V.".
Partnerseiten
Dt. Schulen im Ausland: Mathe-Seiten:

Open Source FunktionenplotterFunkyPlot: Kostenloser und quelloffener Funktionenplotter für Linux und andere Betriebssysteme
Forum "Integrationstheorie" - Doppelintegration
Doppelintegration < Integrationstheorie < Maß/Integrat-Theorie < Analysis < Hochschule < Mathe < Vorhilfe
Ansicht: [ geschachtelt ] | ^ Forum "Integrationstheorie"  | ^^ Alle Foren  | ^ Forenbaum  | Materialien

Doppelintegration: Kann mir jemand bitte helfen?
Status: (Frage) beantwortet Status 
Datum: 17:31 Fr 05.10.2007
Autor: Vogelfaenger

Aufgabe
Lass die Funktion f: [mm] \IR^2 \mapsto \IR [/mm]
f(x, y) = [mm] \bruch{xy}{(x^{2}+y^{2})^2} [/mm]
gegeben sein für (x,y) [mm] \not= [/mm] (0,0)
Mit der konvention f(0,0) = 0

4.1: Ist f integrierbar über das Quadrat [mm] [-1,1]\times[-1,1]? [/mm]

4.2: Zeigen Sie, dass die beiden Doppelintegrale
[mm] \integral_{-1}^{1}{\integral_{-1}^{1}{f(x,y)} dydx} [/mm] und [mm] \integral_{-1}^{1}{\integral_{-1}^{1}{f(x,y)} dxdy} [/mm]
eksistieren.
Sind sie gleich?

Ich habe diese Frage in keinem Forum auf anderen Internetseiten gestellt.

Kann mir bitte jemand erklären, wie ich diese Aufgabe lösen kann?

Das Problem ist, dass ich zu einem Widerspruch komme, indem f offentsichtich sowohl integrierbar als auch nicht-integrierbar ist.

Zu 4.1: Ich muss also prüfen, ob [mm] \integral_{}^{}{|f| dydx} [/mm] < [mm] \infty [/mm]

Die numerische Werte der Funktion heisst also hier, dass das Integral über [mm] [-1,1]\times[-1,1] [/mm] übereinstimmend mit 4 mal das Integral im ersten Quadrant, weil die Funktion symmetrisch ist, das heisst:
[mm] \integral_{-1}^{1}{\integral_{-1}^{1}{ | \bruch{xy}{(x^{2}+y^{2.5})^2}} | dxdy} [/mm] = [mm] 4\integral_{0}^{1}{\integral_{0}^{1}{\bruch{xy}{(x^{2}+y^{2.5})^2}} dxdy} [/mm]

Aber wenn ich diese Werte ausrechne, krieg ich:
[mm] 4\integral_{0}^{1}{\integral_{0}^{1}{\bruch{xy}{(x^{2}+y^{2.5})^{2}}} dxdy} [/mm] = [mm] 2\integral_{0}^{1}{y\integral_{y^2}^{1+y^2}{\bruch{1}{t^{2}}} dtdy} [/mm]
(Substitution: t = [mm] x^2+y^{2}) [/mm]
= [mm] 2\integral_{0}^{1}{y[\bruch{1}{t^{2}}]^{1+y^{2}}_{y^{2}} dy} [/mm] = [mm] 2\integral_{0}^{1}{(\bruch{1}{y} - \bruch{y}{y^{2}+1}) dy} [/mm]
= [mm] 2\integral_{0}^{1}{\bruch{1}{y} dy} [/mm] - [mm] 2\integral_{0}^{1}{\bruch{y}{y^{2}+1} dy} [/mm]
= [mm] 2[lny]^{1}_{0} [/mm] - [mm] \integral_{1}^{2}{\bruch{1}{w} dw} [/mm]

(Substitution: w = [mm] y^{2} [/mm] + 1)

= 2ln1 - [mm] 2\limes_{z\rightarrow\ 0^{+}}lnz [/mm] - [mm] [lnw]^{2}_{1} [/mm] = [mm] \infty [/mm]

Weil das integral unendlich wird, kann die Funktion f(x,y) nicht integrierbar sein, stimmt's?

Aber dann, in 4.2, muss ich zeigen, dass die beide Doppelintegrale existieren.
Aber das heisst ja irgendwie, dass f integrierbar sein muss im Gegensatz zum Ergebnis unter 4.1?

Ich benutze denselben Vorgang, also dass das Integral existiert, wenn [mm] \integral_{}^{}{|f| dydx} [/mm] < [mm] \infty [/mm]

Nehmen wir also das eine innere Integral in einem der beiden Doppelintegrale:
[mm] \integral_{-1}^{1}{|\bruch{xy}{(x^{2}+y^{2})^{2}}| dy} [/mm]

Dann kriegen wir:
[mm] \integral_{-1}^{1}{|\bruch{xy}{(x^{2}+y^{2})^{2}}| dy} [/mm] = [mm] \integral_{-1}^{0}{|\bruch{xy}{(x^{2}+y^{2})^{2}}| dy} [/mm] + [mm] \integral_{0}^{1}{|\bruch{xy}{(x^{2}+y^{2})^{2}}| dy} [/mm]
Weil f in (0,0) nicht kontinuert ist.
Entfernung der numerischen Werten, macht
[mm] \integral_{-1}^{0}{|\bruch{xy}{(x^{2}+y^{2})^{2}}| dy} [/mm] + [mm] \integral_{0}^{1}{|\bruch{xy}{(x^{2}+y^{2})^{2}}| dy} [/mm] = [mm] -x\integral_{-1}^{0}{\bruch{y}{(x^{2}+y^{2})^{2}} dy} [/mm] + [mm] x\integral_{0}^{1}{\bruch{y}{(x^{2}+y^{2})^{2}} dy} [/mm]

Mit der Substitution t = [mm] x^{2}+y^{2} [/mm] kriegt man
= [mm] -\bruch{x}{2}\integral_{x^{2}+1}^{x^{2}}{\bruch{1}{t^{2}} dt} [/mm] + [mm] \bruch{x}{2}\integral_{x^{2}}^{x^{2}+1}{\bruch{1}{t^{2}} dt} [/mm]

= [mm] -\bruch{x}{2}(-\bruch{1}{x^{2}}-(-\bruch{1}{x^{2}+1})) [/mm] + [mm] \bruch{x}{2} (-\bruch{1}{x^{2}+1} [/mm] - [mm] (-\bruch{1}{x^{2}})) [/mm] = [mm] \bruch{1}{x(x^{2}+1)} [/mm]

Und wenn man diesen Ausdruck in das aussere Integral einsetzt und ausrecnet, das heisst
[mm] \integral_{-1}^{1}{\bruch{1}{x(x^{2}+1)} dx} [/mm]
dann kriegt man 0, also etwas endliches statt unendliches und deshalb muss die beiden Integrale existieren?
Aber bin ich nicht eben dazu gekommen, unter 4.1, dass f nicht integriebar war???

Kann jemand bitte die Lösung finden?
Danke.



        
Bezug
Doppelintegration: Antwort
Status: (Antwort) fertig Status 
Datum: 18:32 Fr 05.10.2007
Autor: dormant

Hi!

> Zu 4.1: Ich muss also prüfen, ob [mm]\integral_{}^{}{|f| dydx}[/mm]
> < [mm]\infty[/mm]

Genau.
  

> Die numerische Werte der Funktion heisst also hier, dass
> das Integral über [mm][-1,1]\times[-1,1][/mm] übereinstimmend mit 4
> mal das Integral im ersten Quadrant, weil die Funktion
> symmetrisch ist, das heisst:

Ja, die ist symmetrisch. Aber weil [mm] f(x,y)\rightarrow\infty [/mm] für [mm] (x,y)\rightarrow [/mm] (0,0) sollst du erst über einem Intervall (a,1] mit [mm] a\in [/mm] (0,1) integrieren. Dieser Integral existiert auf jeden Fall. Wenn du dann in dem so erhaltenen Ausdruck für den Integralwert a gegen 0 laufen lässt und etwas kleiner unendlich rauskriegst, so existiert auch der Integral über [0,1]. Dabei ist [mm] a:=(e,f)\in\IR^{2}, [/mm] d.h. du sollst so rechnen:

[mm] \integral_{e}^{1}\integral_{f}^{1}{f(x,y)dxdy}. [/mm]

Dabei kannst du die Betragsstriche weglassen, da auf [mm] f([0,1])\ge [/mm] 0. Und du sollst schön erst nach x (y als Paramter betrachten), dann nach y integrieren. Substitution ist unnötig, du kannst die letzten drei unbestimmten Integrale in dieser []Tabelle benutzen.

>  [mm]\integral_{-1}^{1}{\integral_{-1}^{1}{ | \bruch{xy}{(x^{2}+y^{2.5})^2}} | dxdy}[/mm]
> =
> [mm]4\integral_{0}^{1}{\integral_{0}^{1}{\bruch{xy}{(x^{2}+y^{2.5})^2}} dxdy}[/mm]
>  
> Aber wenn ich diese Werte ausrechne, krieg ich:
>  
> [mm]4\integral_{0}^{1}{\integral_{0}^{1}{\bruch{xy}{(x^{2}+y^{2.5})^{2}}} dxdy}[/mm]
> =
> [mm]2\integral_{0}^{1}{y\integral_{y^2}^{1+y^2}{\bruch{1}{t^{2}}} dtdy}[/mm]
>  
> (Substitution: t = [mm]x^2+y^{2})[/mm]
>  = [mm]2\integral_{0}^{1}{y[\bruch{1}{t^{2}}]^{1+y^{2}}_{y^{2}} dy}[/mm]
> = [mm]2\integral_{0}^{1}{(\bruch{1}{y} - \bruch{y}{y^{2}+1}) dy}[/mm]
>  
> = [mm]2\integral_{0}^{1}{\bruch{1}{y} dy}[/mm] -
> [mm]2\integral_{0}^{1}{\bruch{y}{y^{2}+1} dy}[/mm]
>  = [mm]2[lny]^{1}_{0}[/mm]
> - [mm]\integral_{1}^{2}{\bruch{1}{w} dw}[/mm]
>  
> (Substitution: w = [mm]y^{2}[/mm] + 1)
>  
> = 2ln1 - [mm]2\limes_{z\rightarrow\ 0^{+}}lnz[/mm] - [mm][lnw]^{2}_{1}[/mm] =
> [mm]\infty[/mm]
>  
> Weil das integral unendlich wird, kann die Funktion f(x,y)
> nicht integrierbar sein, stimmt's?

Eigentlich schon. Du könntest dich verrechnet haben.
  

> Aber dann, in 4.2, muss ich zeigen, dass die beide
> Doppelintegrale existieren.
>  Aber das heisst ja irgendwie, dass f integrierbar sein
> muss im Gegensatz zum Ergebnis unter 4.1?

Ja. 4.1 und 4.2 ist so ziemlich die selbe Aufgabe.
  

> Ich benutze denselben Vorgang, also dass das Integral
> existiert, wenn [mm]\integral_{}^{}{|f| dydx}[/mm] < [mm]\infty[/mm]
>  
> Nehmen wir also das eine innere Integral in einem der
> beiden Doppelintegrale:
>  [mm]\integral_{-1}^{1}{|\bruch{xy}{(x^{2}+y^{2})^{2}}| dy}[/mm]
>  
> Dann kriegen wir:
>  [mm]\integral_{-1}^{1}{|\bruch{xy}{(x^{2}+y^{2})^{2}}| dy}[/mm] =
> [mm]\integral_{-1}^{0}{|\bruch{xy}{(x^{2}+y^{2})^{2}}| dy}[/mm] +
> [mm]\integral_{0}^{1}{|\bruch{xy}{(x^{2}+y^{2})^{2}}| dy}[/mm]
>  Weil
> f in (0,0) nicht kontinuert ist.
>  Entfernung der numerischen Werten, macht
>  [mm]\integral_{-1}^{0}{|\bruch{xy}{(x^{2}+y^{2})^{2}}| dy}[/mm] +
> [mm]\integral_{0}^{1}{|\bruch{xy}{(x^{2}+y^{2})^{2}}| dy}[/mm] =
> [mm]-x\integral_{-1}^{0}{\bruch{y}{(x^{2}+y^{2})^{2}} dy}[/mm] +
> [mm]x\integral_{0}^{1}{\bruch{y}{(x^{2}+y^{2})^{2}} dy}[/mm]
>  
> Mit der Substitution t = [mm]x^{2}+y^{2}[/mm] kriegt man
>  =
> [mm]-\bruch{x}{2}\integral_{x^{2}+1}^{x^{2}}{\bruch{1}{t^{2}} dt}[/mm]
> + [mm]\bruch{x}{2}\integral_{x^{2}}^{x^{2}+1}{\bruch{1}{t^{2}} dt}[/mm]
>  
> = [mm]-\bruch{x}{2}(-\bruch{1}{x^{2}}-(-\bruch{1}{x^{2}+1}))[/mm] +
> [mm]\bruch{x}{2} (-\bruch{1}{x^{2}+1}[/mm] - [mm](-\bruch{1}{x^{2}}))[/mm] =
> [mm]\bruch{1}{x(x^{2}+1)}[/mm]
>  
> Und wenn man diesen Ausdruck in das aussere Integral
> einsetzt und ausrecnet, das heisst
>  [mm]\integral_{-1}^{1}{\bruch{1}{x(x^{2}+1)} dx}[/mm]
>  dann kriegt
> man 0, also etwas endliches statt unendliches und deshalb
> muss die beiden Integrale existieren?
>  Aber bin ich nicht eben dazu gekommen, unter 4.1, dass f
> nicht integriebar war???
>  
> Kann jemand bitte die Lösung finden?
>  Danke.

Gruß,
dormant

Bezug
                
Bezug
Doppelintegration: Frage (beantwortet)
Status: (Frage) beantwortet Status 
Datum: 12:38 Sa 06.10.2007
Autor: Vogelfaenger

Ok, danke für die Antwort

Aber wenn ich das tue, komm ich immernoch nicht duch.

Ich habe also dann
[mm] \integral_{e}^{1}\integral_{f}^{1}{\bruch{xy}{(x^{2}+y^{2})^{2}}dxdy} [/mm] = [mm] \integral_{e}^{1}{y\integral_{f}^{1}{\bruch{x}{(x^{2}+y^{2})^{2}}}dxdy} [/mm]
Mit Limes und dem unbestimmten Integrale aus der Tabelle wird es dann:
[mm] \integral_{e}^{1}\limes_{f\rightarrow\ 0^{+}}{y\integral_{f}^{1}{\bruch{x}{(x^{2}+y^{2})^{2}}}dxdy} [/mm] = [mm] \integral_{e}^{1}{\limes_{f\rightarrow\ 0^{+}}y\integral_{f}^{1}{\bruch{1}{2(x^{2}+y^{2})}dx} dy} [/mm]

= [mm] \integral_{e}^{1}{y\limes_{f\rightarrow\ 0^{+}}(\bruch{1}{2(1+y^{2})} - \bruch{1}{2(f^{2}+y^{2})}) dy} [/mm] = [mm] \integral_{e}^{1}{\bruch{1}{2y(1+y^{2})}dy} [/mm]

Und dann bekommen wir das selbe Problem wir früher.

> >   Du könntest dich verrechnet haben.

>    
>

Ja, eben ;-)
Aber weiss jemand vielleicht genau wo der Fehler liegt?


Bezug
                        
Bezug
Doppelintegration: Antwort
Status: (Antwort) fertig Status 
Datum: 23:34 Sa 06.10.2007
Autor: leduart

Hallo
im ersten Teil hast du richtig bewiesen, dass das Integral auf dem gegebenen Intervall nicht lesbegues integrierbar ist.
Deshalb ist der zweite Teil natürlich gar nicht lesbegues integrierbar und du musst nichts mehr tun.
Wenn du dagegen das Riemannintegral bildest, ist der Beitrag im 1.ten und 3.ten Quadranten [mm] +\infty, [/mm] im 2ten und 4 ten [mm] -\infty, [/mm] und wenn du ohne GW Betrachtung rechnest hast du eben [mm] +\infty [/mm] + [mm] -\infty=0. [/mm]
Ich denke das richtige Vorgehen wäre, einen [mm] \varepsilonkreis [/mm] um 0 wegzulassen. aussymmetriegründen hebt sich dann alles ausserhalb des Kreises Weg, d.h. das Integral über [mm] [-1,+1]\times [-1,+1]/x^2+y^2<\varepsilonquadrat [/mm] ist 0.
dann bildest du den GW für [mm] \varepsilon [/mm] gegen 0 und hast die 0.
das liese ich als ergebnis stehen.
Das ist nicht der Wert des Integrals, sondern ein GW im eindimensionalen heisst er Cauchyscher Hauptwert . z.Bsp ist das Integral über 1/x von -1 bis +1 nicht lesbueges integrierbar, aber sein cauchyscher hauptwert ist 0.
siehe []wiki
Gruss leduart
Wie das bei 2 dim ist, weiss ich nicht.

Bezug
Ansicht: [ geschachtelt ] | ^ Forum "Integrationstheorie"  | ^^ Alle Foren  | ^ Forenbaum  | Materialien


^ Seitenanfang ^
www.vorhilfe.de